4
$\begingroup$

Let $(R, \mathfrak{m})$ be a noetherian local ring, and let $\hat R$ be its $\mathfrak{m}$-adic completion. Extension of scalars allows one to transform an $R$-scheme into an $\hat R$-scheme. Is this association surjective, up to isomorphism? More precisely, given a finite type scheme $X \to \mathrm{Spec} \ \hat R$, does there exist an $R$-scheme $Y$ and an $\hat R$-isomorphism $$X \cong Y \times_{\mathrm{Spec} \ R} \mathrm{Spec} \ \hat R\ \ ? $$

My interest stems from the case where $R$ is the ring of germs of holomorphic functions at the origin in $\mathbb{C}$. If $t$ is the usual coordinate on $\mathbb{C}$, then we have a canonical isomorphism $\hat R \cong \mathbb{C}[[t]]$, where the latter is the ring of formal power series in $t$. Is every formal family $X \to \mathbb{C}[[t]]$ isomorphic to an analytic family? I suppose it may be relevant that $R$ is henselian in my example (by the implicit function theorem).

Update: Let $K$ be the field of fractions of $R$, and let $\hat K$ be the field of fractions of $\hat R$. Suppose that there exists a $K$-scheme $Y_K$ and a $\hat K$ isomorphism $X_{\hat K} \cong Y_K \times_{\mathrm{Spec} \ K} \mathrm{Spec \ \hat K}$. Can we conclude that there exists an $R$-scheme $Y$ as above, which necessarily has generic fiber $Y_K$? That is, if we already know the generic fiber is algebraic, can we arrange for the whole scheme to be algebraic?

$\endgroup$
9
  • 8
    $\begingroup$ An elliptic curve over $\widehat{R}$ with $j$-invariant not in $R$ is a counterexample. (Concretely, if the $j$-invariant over $\mathbf{C}[\![t]\!]$ has radius of convergence 0 then it cannot arise from an analytic family of elliptic curves.) $\endgroup$
    – user30180
    Jun 15, 2013 at 0:14
  • 1
    $\begingroup$ For any noetherian pair $(R,\mathfrak{m})$, one knows: giving a quasicoherent (qc) sheaf $F$ on $S := \mathrm{Spec}(R)$ is the same as giving a qc sheaf $\hat{F}$ on $\hat{S} := \mathrm{Spec}(\hat{R})$, a qc sheaf $F_U$ on $U = S - V(\mathfrak{m})$, and an isomorphism between the two over $U \times_S \hat{S}$. The same comment applies to "quasicoherent sheaves" replaced by "affine/quasiprojective morphisms." $\endgroup$
    – anon
    Jun 15, 2013 at 8:51
  • 1
    $\begingroup$ stacks.math.columbia.edu/tag/05E5 $\endgroup$
    – anon
    Jun 15, 2013 at 9:25
  • 1
    $\begingroup$ @ayanta: Thanks; of course, I meant (and should have said) to keep track of the ample line bundle. How does one get around having to worry about $\hat{R} \otimes_R \hat{R}$ while directly applying fpqc descent? $\endgroup$
    – anon
    Jun 15, 2013 at 13:28
  • 1
    $\begingroup$ @ayanta: Doesn't the descent datum, at least a priori, entail specifying additional data over $\hat{R} \otimes_R \hat{R}$ (satisfying a cocycle condition) in addition to the isomorphism over $K \otimes_R \hat{R}$? $\endgroup$
    – anon
    Jun 15, 2013 at 16:17

2 Answers 2

5
$\begingroup$

This is really just ayanta's example again: even with the condition on the generic fiber, the answer is still "no". Let $R$ be the local ring of $k[s,t]$ at the maximal ideal $\langle s,t\rangle$. Begin with $Y_R$ being $\mathbb{P}^1_R$ -- this is not the final scheme. For the base change $Y_{\widehat{R}}$ of $Y_R$, restrict over the closed subscheme $Z(s) = \text{Spec} \widehat{R}/s\widehat{R}$. Let $D_s \subset \mathbb{P}^1_{Z(s)}$ be a Cartier divisor that is flat over $Z(s)$ of relative degree $4$ and whose corresponding $j$-invariant, $j(D_s)\in \widehat{R}/s\widehat{R}$, is not the pullback of an element in $R/sR$. Now let $X$ be the blowing up of $\mathbb{P}^1_{\widehat{R}}$ along the codimension $2$ closed subscheme $D_s$.

$\endgroup$
1
  • $\begingroup$ Great example! If I understand correctly, the fact that $R$ has dimension $>1$ means that you can arrange a descent obstruction without touching the generic fiber of $\mathbb{P}^1_R$. $\endgroup$ Jun 19, 2013 at 1:36
7
$\begingroup$

In Jason's example, the obstruction is that even the restriction above $\mathrm{Spec}(\widehat{R}/s\widehat{R})$ does not descend. Now let $U\subset \mathrm{Spec}({R})$ (resp. $\widehat{U}\subset \mathrm{Spec}(\widehat{R})$) be the complement of the closed point. If one assumes that the restriction of $X$ to $\widehat{U}$ descends to $U$, then the answer is yes in many cases. This is discussed in my paper "Un problème de descente", Bull. Soc. Math. France 124 (1996), 559-585.

Of course, in Xander's original case, we have $U=\mathrm{Spec}(K)$.

Unfortunately I have no time right now to elaborate (or even check the statements!). If I remember correctly, there is a descent datum on $X$ relative to $R\to\widehat{R}$, so the problem then becomes an effectivity question.

$\endgroup$
1
  • $\begingroup$ @Laurent Many thanks! This article is exactly what I need! $\endgroup$ Jun 19, 2013 at 1:32

Your Answer

By clicking “Post Your Answer”, you agree to our terms of service and acknowledge you have read our privacy policy.

Not the answer you're looking for? Browse other questions tagged or ask your own question.